2
$\begingroup$

I'm trying to find a proof of this counterexample by von Neumann:

Let $x_{mn}\in \ell^2$ be defined by $$x_{mn}(m)=n \quad,\quad x_{mn}(n)=m \quad,\quad x_{mn}(k)=0 \hbox{ otherwise,} $$ and let $S=\{ x_{mn} : m, n\geq 1\}$. Von Neumann shows that $0$ is in the weak closure of this set but no sequence in $S$ convergess weakly to $0$.

$\endgroup$
3
  • 1
    $\begingroup$ I have taken the liberty of rewriting and retitling the question, hopefully preserving the original sense. That said, I am not sure if the question really belongs on MO $\endgroup$
    – Yemon Choi
    Mar 14, 2012 at 21:30
  • 1
    $\begingroup$ @YemonChoi I believe the correct formulation is $x_{mn}(m) = 1, x_{mn}(n) = m,$ and $x_{mn}(k) = 0$ if $k \neq m,n$. Source: "A Course in Functional Analysis" by Conway Chapter 5.13 Question 2. $\endgroup$ Apr 15, 2021 at 23:54
  • $\begingroup$ @mathnoob123 Thank you. To be honest I am not sure it is worth bumping the question just to correct this, especially as the answers below seem to have correctly identified what is going on, and since the person who asked this question has not been seen on this site for 9 years. But thank you for tracking down a corrected formulation. $\endgroup$
    – Yemon Choi
    Apr 16, 2021 at 5:38

2 Answers 2

6
$\begingroup$

As Aaron pointed out, "von Neumann's" example is really a non example. To salvage the problem, restate it as: construct a sequence in $\ell_2$ which has $0$ in its weak closure, but no subsequence converges weakly to $0$. First note that such a sequence must be unbounded (by Eberlein-Smulian). Secondly, observe that it is enough to have for each $\epsilon > 0$ a (necessarily bounded) subsequence that converges weakly to a point whose norm is at most $\epsilon$ (and, of course, no subsequence that converges weakly to $0$). With these "hints", it is easy to construct an example: Let $x_{nm}(k)$ be $1/n$ if $k=1$, $n$ if $k=m>1$, and $0$ otherwise. With the "obvious" definition, $0$ is in the $2$-weak sequential closure of $x_{nm}$ but not in the $1$-weak sequential closure. From this beginning it is natural to define for each countable ordinal $\alpha$ the $\alpha$-weak sequential closure and to state an obvious problem. Another (not very difficult once you understand the example above) problem is to build a sequence in $\ell_2$ whose norms tend to infinity and yet $0$ is in the weak closure of the sequence.

Another striking example of the phenomena sought by the OP is the following. Take a dense sequence in the unit sphere of $\ell_1$. Then $0$ is in the weak closure of the sequence but no subsequence converges weakly to $0$ because $\ell_1$ has the Shur property.

$\endgroup$
2
  • $\begingroup$ Did you mean "unit sphere of $\ell_1$"? $\endgroup$ Mar 15, 2012 at 20:03
  • $\begingroup$ Another small typo: I think in the definition of $x_{nm}(k)$, you want $n=m > 1$ to read $k=m > 1$. $\endgroup$ Mar 15, 2012 at 21:21
5
$\begingroup$

I don't believe that $0$ is a weak cluster point of this set. For example, consider $y \in \ell^2$ defined by $$y(k) = 1/k.$$ Then we have, for any $m,n$ that $$\langle x_{m,n}, y \rangle = m/n + n/m \geq 2.$$ Therefore, the weak neighbourhood $$ \{x \in \ell^2: |\langle x, y\rangle| < 1\} $$ of $0$ does not intersect $S$.

$\endgroup$
1
  • $\begingroup$ It is possible that I made an error in transcription when reformatting the original question, but unless the OP turns up thete's no way of knowing $\endgroup$
    – Yemon Choi
    Mar 15, 2012 at 18:25

Your Answer

By clicking “Post Your Answer”, you agree to our terms of service and acknowledge you have read our privacy policy.

Not the answer you're looking for? Browse other questions tagged or ask your own question.